Thread Rating:
  • 0 Vote(s) - 0 Average
  • 1
  • 2
  • 3
  • 4
  • 5
NBME Ob/Gyn Form 4 questions - spirohnolactone
#1
1) A 52-year-old woman comes to the physician for a routine examination. She has mild hypothyroidism well controlled with levothyroxine. Five years ago, she was diagnosed with stage I breast cancer; she completed tamoxifen therapy 1 year ago and is currently in remission. Menopause occurred 4 years ago Her mother sustained a femoral fracture in a bicycle collision at the age of 55 years. The patient has smoked two cigarettes weekly for 25 years. She is 150 cm (4 ft 11 in) tall and weighs 43 kg (95 1b); BMI is 19 kg/m2. Examination shows no thyromegaly. There is a small; well-healed surgical scar over the upper outer quadrant of the left breast
The remander of the breast and pelvic examinations show no abnormalities. Which of the following historical findings is the greatest risk factor for osteoporotic fracture in this patient?

A) BMI
B) Family history of fracture (incorrect)
C) Levothyroxine therapy
D) Tamoxifen therapy
E) Tobacco use

2) A 57-year-old woman, gravida 3, para 3, comes to the physician because of loss of urine for 2 weeks She now wears an absorbent pad constantly and frequently soaks the pad. All of her children were delivered vaginally with no complications. Three years ago; she received radiation therapy for cervical cancer and has been cancer free since then. She has no other history of serious illness and takes no medications. Vital signs are within normal limits. Physical examination shows no abnormalities. Genitourinary examination shows thin, atrophic, moist vaginal mucosa; there is a pool of fluid in the vaginal canal. Her postvoid residual volume is less than 10 mL. Urinalysis shows no abnormalities. Which of the following is the most likely diagnosis?

A) Overactive bladder
B) Overflow incontinence
C) Stress incontinence (incorrect)
D) Urinary retention
E) UTI
F) Vesicovaginal fistula

3) A 32-year-o d woman, gravida 1, para 1, at 34 weeks' gestation is admitted to the hospital in labor. Pregnancy had been complicated by blurred vision and a 23-kg (5-1b) weight gain over the past week. Her last routine prenatal Visit 1 month ago showed no abnormalities. Her temperature is 37 oc (98.6'F), pulse IS 80/min, respirations are 20/min, and blood pressure is 150/110 mm Hg. Examination shows retinal arteriolar spasms and pedal edema Laboratory studies show:

Hemoglobin 12.5 g/dL
Leukocyte count 8000/mm^3
Platelet count 200,000/mm^3
Serum creatinine 1.2 mg/dL
Urine protein 2+

This patient's fetus is at greatest risk for which of the following obstetric complications?

A) Brachial plexus injury
B) Fetal growth restriction
C) Hydrops
D) Macrosomia (incorrect)
E) Pulmonary hypoplasia
F) Sepsis

4) A widowed 37-year-old woman, gravida 3, para 3, whose youngest ch Id is 10 years old; has had increasingly heavy but regular menstrual periods for the past 3 years. Her last menses occurred 21/2 weeks ago. She reports a 2 day history of labor-like pains accompanied by a small amount of vaginal bleeding. Speculum examination shows that the cervix is moderately effaced and 2 cm dilated, with some beefy red tissue at the os. Which of the following is the most likely diagnosis?

A) Carcinoma of the cervix
B) Carcinoma of the endometrium
C) Incomplete abortion (incorrect)
D) Pedunculated submucous leiomyoma uteri
E) Sarcoma of the uterus

5) A 36-year-old woman, gravida 2, para 1, at 41 weeks' gestation has had ruptured membranes without contractions for 8 hours. Her first infant weighed 4422 g (9 1b 12 oz) at birth. This pregnancy has been uncomplicated except for gestational diabetes, which was diagnosed at 26 weeks' gestation and has been well controlled With diet. Initial assessment shows a fundal height of 40 cm. On ultrasonography, the estimated fetal weight is 3714 g (8 lb 3 oz) The cerv x IS 2 cm dilated and 50% effaced. The fetal heart rate is within normal limits. Labor is induced with intravenous oxytocin. Four hours later, her cervix is 4 cm dilated and completely effaced. Continuous epidural anesthesia is administered. Two hours later, the fetal heart rate pattern demonstrates late decelerations with each contraction. The contractions occur every minute, last 45 seconds, and are 75 mm Hg at their peak. Which of the following is the most likely explanation for this pattern?

A) Epidural anesthesia
B) Fetal macrosomia
C) Gestational diabetes (incorrect)
D) Oxytocin administration
E) Postdates pregnancy

6) Two days after a cesarean delivery for fetal distress, a 27-year-old woman has temperatures to 38.2'C (100.8 F) She had an 8-hour labor complicated by mild meconium-stained amniotic fluid. Membranes ruptured 2 hours after admission. Breath sounds are decreased at both lung bases. Abdominal examination shows mild tenderness below the umbilicus and a well-healing incision. There is a moderate amount of lochia with no odor. Examination of the lower extremities shows edema and no tenderness. Her hemoglobin concentration is 10 8 g/dL, leukocyte count is 14000/mm3, and platelet count is 189,000/mm3. A urine catheter specimen is negative.

Which of the following is the most likely diagnosis?

A) Atelectasis
B) Deep venous thrombosis
C) Endometritis
D) Episiotomy infection
E) Mastitis
F) pneumonia
G) Pulmonary embolism (incorrect)
H) Pyelonephritis
l) Septic pelvic thrombophlebitis
J) Ureteral obstruction
K) Urinary tract infection

7) A 36-year-o d woman, gravida 1, para 1, has been unable to conceive for 1 year. Her vaginal delivery 2 years ago was complicated by postpartum hemorrhage and endometritis treated With dilatation and curettage. She has not resumed menses since delivery but does have cyclic abdominal pain. Examination shows normal findings. Her husband's semen analysis shows normal findings. Which of the following is the most likely cause?

A) Anovulation (incorrect)
B) Antisperm antibodies
C) Endometriosis
D) Luteal phase defect
E) Periadnexal adhesions
F) Tubal occlusion
G) Uterine synechiae

8) A 1 5-year-old girl is brought to the physician because of a 3-day history of fever, abdominal pain, and nausea. She also has had a thick; white vaginal discharge. Menarche was at the age of 13 years; and her menses occur at irregular 28- to 40-day intervals. She became sexually active 1 month ago and uses condoms inconsistently. Her temperature is 39.6C (103.2F); pulse is 108/min, respirations are 20/min, and blood pressure is 102/80 mm Hg. Exam nation shows lower abdominal tenderness. Pelvic examination shows pain with cervical motion and adnexal tenderness With a 3-cm mass. A Gram stain of vaginal discharge shows gram-negative diplococci.

Which of the following is the most likely explanation for this patient's susceptibility to this condition?

A) Immature immune system (incorrect)
B) Increased cervical cell vulnerability to infection
C) Increased concentrations of progesterone
D) Latex hypersensitivity
E) Menarche before 14 years of age
Reply
#2
Sorry, for #6, meant to say Pneumonia was incorrect. Not sure about PE.
Reply
#3
1a2f3e4d5e6a7g8d.
Reply
#4
Please correct if am wrong
Reply
#5
1 i put B which was wrong. It is probably D
http://www.ncbi.nlm.nih.gov/pmc/articles/PMC3720571/
Reply
#6
by D I mean C
Reply
#7
@luciphear why not Tamoxifen as your probability at your first post pick.
Reply
#8
tamoxifen is protective for osteoporosis
http://www.ncbi.nlm.nih.gov/pmc/articles/PMC4438570/

also on picmonic: http://d1abqebhfcygq8.cloudfront.net/clo...n_v1.6.jpg
Reply
#9
2 i got this wrong - didn't read it enough - should be f - her post void volume is 10 which is
Reply
#10
ugh my work got erased.

2 f
4 d
5 a
6 a

for sure:
3 b
7 g
8 e
Reply
« Next Oldest | Next Newest »


Forum Jump: